Author Topic: EM Drive Developments - related to space flight applications - Thread 3  (Read 3130740 times)

Offline Notsosureofit

  • Full Member
  • ****
  • Posts: 691
  • Liked: 747
  • Likes Given: 1729

Please restudy the GR! The planklenght is dependent on the strength of G The light have to travel a longer way. So it is not faster or slower! Inside a Material the permittivity and permabillity is higher than in vacuum

or got anybody a other universal meter in the toolbox?

The Plank length is defined in terms of the Gravitational constant "G", not the frame acceleration "g".  The only variable in that definition is the speed of light "c".

Offline phaseshift

  • Full Member
  • *
  • Posts: 104
  • Seattle, WA
  • Liked: 84
  • Likes Given: 97
As many have said before the EW drive just can't work without violating CoM.  The frustum is a closed cavity, at some level the forces must balance out. Unless, somehow, the frustum is being 'tricked' into thinking it's actually open.

Shawyer attempts to use Special Relativity to open the system, White uses the Quantum Vacuum, what is Yang proposing? Yang talks about charged particles but that doesn't fit the bill.

"It doesn't have to be a brain storm, a drizzle will often do" - phaseshift

Offline WarpTech

  • Full Member
  • ****
  • Posts: 1407
  • Do it!
  • Statesville, NC
  • Liked: 1453
  • Likes Given: 1925
...A distant observer may see these geodesics as curved, but nevertheless when a particle follows a null geodesic, it is not accelerating in the GR sense.
http://physics.stackexchange.com/questions/20289/do-photons-have-acceleration/20296#20296

From our perspective as distant observers of what is happening inside the frustum, light propagating in an accelerated references frame is accelerated. If it were not so, gravitational lensing would not be possible. If you want to work in the "local" frame, light always travels at c, space-time is flat, Lorentz transformations apply and the speed of light is Lorentz Invariant. The coordinate speed I am talking about is not the invariant speed in the sense of Lorentz invariance. Lorentz invariance is not applicable in accelerated frames, and I am not talking about the "local" speed of light on a tangent plane. I am talking about comparing the speed of light at one gravitational potential with the speed at another.

Regarding K, it is the refractive index of the Polarizable Vacuum Model. I've posted this enough times already that I no longer feel obligated to define it again every time I use it.
EDIT:
ds^2 = -g_00*c^2*dt^2 + g_11*dx^2 = 0   Null line element

dx/dt = c*sqrt(-g_00/g11), coordinate speed of light = c/K

K = sqrt(-g11/g00)

K = 1/(1 - 2GM/r*c^2)

In a gravitational field according to Schwarzschild, where M is the mass of the planet, and r is the radius from the Center of Mass. This is approximated as;

K ~ exp[2GM/r*c^2]

Therefore: c/K depends on your altitude in a gravitational field such that the coordinate speed of light in a gravitational field OR an accelerated reference frame is NOT constant. In the case of the frustum,

c => w^2/(a^2 + k^2) = c/K, where w is frequency and "a" is the attenuation factor. In fact, I will go one step farther and claim that the refractive index due to gravity is "in fact" due to attenuation of the quantum wave functions of the harmonic oscillators that describe matter, all the way down to the Quarks! It's the same thing.

http://www.earthtech.org/publications/PV_Found_of_Physics.pdf


Todd
« Last Edit: 06/02/2015 06:49 pm by WarpTech »

Good day all!

Please, a bit of leeway here. This is my first post on this forum, and considering the importance of what is being discussed I have absolutely no intention of wasting anyone's time. With that said, I will continue on to say that Mathematics is certainly not my strong point.

I do however have a good "Feel" for these things, and a base understanding to boot.

I would like to posit a question. And quite a difficult one at that. Please excuse my ignorance at the moment as I am quite rusty and have forgotten quite a few things.

Can a virtual particle impart information to it's surroundings within a vacuum? (within string theory?)

Now I know I probably phrased that incorrectly, but work with me here... I might have an idea what might be going on here. I think I see a problem with the way people are looking at the relative framework...

Offline WarpTech

  • Full Member
  • ****
  • Posts: 1407
  • Do it!
  • Statesville, NC
  • Liked: 1453
  • Likes Given: 1925
As many have said before the EW drive just can't work without violating CoM.  The frustum is a closed cavity, at some level the forces must balance out. Unless, somehow, the frustum is being 'tricked' into thinking it's actually open.

Shawyer attempts to use Special Relativity to open the system, White uses the Quantum Vacuum, what is Yang proposing? Yang talks about charged particles but that doesn't fit the bill.

It's not a closed cavity. It has a hole in it where microwave energy is being fed into it. This means there is positive Divergence. If you turn off the source and plug the hole, it is a closed cavity which has zero Divergence. If you turn it off, there is no thrust.

Todd



Offline Rodal

  • Senior Member
  • *****
  • Posts: 5911
  • USA
  • Liked: 6124
  • Likes Given: 5564
Nobel Prize winner Hans Bethe showed more than 70 years ago that the Poynting vector of the field through a hole in  a cavity (the RF feed) is non-zero

Look at Equations 45 through 47 and also 54.

If RF power is being fed into a cavity, that constitutes a source into the cavity, power is being fed through the orifice and this must involve a non-zero Poynting vector.

Bethe's equation for power coupling of a waveguide to a cavity are of intrinsic value to the design of cavities at particle accelerators to this day.

« Last Edit: 06/02/2015 06:44 pm by Rodal »

Offline phaseshift

  • Full Member
  • *
  • Posts: 104
  • Seattle, WA
  • Liked: 84
  • Likes Given: 97
As many have said before the EW drive just can't work without violating CoM.  ...
Who has said that "the EM Drive drive just can't work without violating CoM"? I don't know of any author that has advocated that the EM Drive works by violating Conservation of Momentum.  Every single theory I know of, Shawyer, McCulloch, Yang, White, you name it, postulates that the EM Drive does not violate conservation of momentum (they all have different explanations as to why momentum is conserved).

I think you can find the forum is full of extremely knowledgeable people who have said as such.  Yes, the authors you listed have made claims but in each case have opened their system such that CoM appears to not be violated and that's how they make the 'claim' they are not violating basic physics.

Focusing on the question I asked might be more productive.  How did Yang open her system such that CoM does not appear to be violated?  Or do you really believe that the EM Drive, a closed cavity, is not violating CoM without a mechanism for opening it up and therefore in your opinion this line of questioning can lead no where?
"It doesn't have to be a brain storm, a drizzle will often do" - phaseshift

Offline SeeShells

  • Senior Member
  • *****
  • Posts: 2442
  • Every action there's a reaction we try to grasp.
  • United States
  • Liked: 3186
  • Likes Given: 2708
As many have said before the EW drive just can't work without violating CoM.  ...
Who has said that "the EM Drive drive just can't work without violating CoM"? I don't know of any author that has advocated that the EM Drive works by violating Conservation of Momentum.  Every single theory I know of, Shawyer, McCulloch, Yang, White, you name it, postulates that the EM Drive does not violate conservation of momentum (they all have different explanations as to why momentum is conserved).
I think you can find the forum is full of extremely knowledgeable people who have said as such.  Yes, the authors you listed have made claims but in each case have opened their system such that CoM appears to not be violated and that's how they make the 'claim' they are not violating basic physics.

Focusing on the question I asked might be more productive.  How did Yang open her system such that CoM does not appear to be violated?  Or do you really believe that the EM Drive, a closed cavity, is not violating CoM without a mechanism for opening it up and therefore in your opinion this line of questioning can lead no where?

Even the most disruptive force in our known universe, a black hole, it has to play by the rules and not violate CoM and CoE, drop down to plank scale you might pull something from the quantum foam, but only for the briefest amount of time. Even SpaceTime when inflation  ruled and everything accelerated away exponentially from everything else, still didn't violate CoE and CoM. To me a fundamental fact, is you can't do it. Just because it looks like the drive is a closed container it doesn't mean it really is, does it? Somehow that mix of RF bouncing around in the copper Frustum is making it to the outside world or an outside force is felt through the copper walls and the wave functions are acting on it.
Data, we need more Data, to be able to fill in the theories.

Offline deltaMass

  • Full Member
  • ****
  • Posts: 955
  • A Brit in California
  • Liked: 671
  • Likes Given: 275
In conventional physics, it is clear that the momentum of the radiation injected into the cavity produces a back reaction upon the source of that radiation. Therefore if you put a box around [source + cavity], where the source includes a portable power supply (e.g. battery) - then the net momentum of the system is zero forever as viewed from outside the box.

If there is a radiation leak, thrust no greater than P/c ("equivalent photon rocket") may result from such a system, including thermal effects originating from the input power P.

Clearly here the claim being made is not conventional physics because:

1. The claim is made that the box as a whole will move
and
2. The claim is made that the magnitude of the thrust causing this movement exceeds the maximum expected thrust of the equivalent photon rocket by orders of magnitude.

Thus any successful attempt at explanation will not use conventional physics.


There is a corollary to all this:

If you attempt to explain this effect with conventional physics alone, you have made an error.
« Last Edit: 06/02/2015 06:54 pm by deltaMass »

Offline phaseshift

  • Full Member
  • *
  • Posts: 104
  • Seattle, WA
  • Liked: 84
  • Likes Given: 97
As many have said before the EW drive just can't work without violating CoM.  ...
Who has said that "the EM Drive drive just can't work without violating CoM"? I don't know of any author that has advocated that the EM Drive works by violating Conservation of Momentum.  Every single theory I know of, Shawyer, McCulloch, Yang, White, you name it, postulates that the EM Drive does not violate conservation of momentum (they all have different explanations as to why momentum is conserved).

I think you can find the forum is full of extremely knowledgeable people who have said as such.  Yes, the authors you listed have made claims but in each case have opened their system such that CoM appears to not be violated and that's how they make the 'claim' they are not violating basic physics.

Focusing on the question I asked might be more productive.  How did Yang open her system such that CoM does not appear to be violated?  Or do you really believe that the EM Drive, a closed cavity, is not violating CoM without a mechanism for opening it up and therefore in your opinion this line of questioning can lead no where?
Perhaps we are using different languages to express this.

I use the following definition of an open system:

"A open system is a system that has external interactions. Such interactions can take the form of information, energy, or material transfers into or out of the system boundary, depending on the discipline which defines the concept. An open system is contrasted with the concept of an isolated system which exchanges neither energy, matter, nor information with its environment. An open system is also known as a constant volume system or a flow system."

According to that definition Dr. White's theory is opening the system by involving the Quantum Vacuum, if we take that the Quantum Vacuum was not part of the system being considered.

 I think that neither Shawyer, McCulloch or Yang are "opening the system".    I don't think that discussing a gradient of group velocity, or Unruh radiation, or considering current density J is opening the system. But I guess that it all depends on what one means by an open system.

As to what Prof. Yang is doing, I don't completely understand it, but my take (with a grain of salt) is that she maybe considering the case of a cavity coupled externally with a waveguide or a coaxial. Thus, the excitations of a mode in a cavity can be modeled by an equivalent electric ( J ) or magnetic (  Jm ) density current representing the sources of the modes. The equivalent magnetic sources are, for example, the magnetic field on a coupling slot between the waveguide and the cavity and the magnetic field generated by a loop coupled with a cavity, while the equivalent electric sources are the currents on a small antenna coupled with the cavity.

We are using the same definition of open system :)

I do think Shawyer  attempted to "open" the cavity by invoking Special Relativity (incorrectly). White uses the QV (almost in an extra dimensional way) to open the system. Yang appears to make no attempt at doing so and thus the reason for my question.

Going extra dimensional, 4+1, also opens up the cavity. However, in looking over Randall/Sundrum I realized that their +1 is not on our D-Brane and is rather the bulk itself.  According to string theory this means RF energy can not enter the +1 dimension,yet  in the same breath Randall wonders if Standard Model particles are in the bulk.  Of course there is the possibility of other dimensions outside of their theory.

I am hoping that given Yang's substantially higher thrust that the manner in which the cavity is opened up could be discerned helping theory move forward.

"It doesn't have to be a brain storm, a drizzle will often do" - phaseshift

Offline Rodal

  • Senior Member
  • *****
  • Posts: 5911
  • USA
  • Liked: 6124
  • Likes Given: 5564
In conventional physics, it is clear that the momentum of the radiation injected into the cavity produces a back reaction upon the source of that radiation. Therefore if you put a box around [source + cavity], where the source includes a portable power supply (e.g. battery) - then the net momentum of the system is zero forever as viewed from outside the box.

If there is a radiation leak, thrust no greater than P/c ("equivalent photon rocket") may result from such a system, including thermal effects originating from the input power P.

Clearly here the claim being made is not conventional physics because:

1. The claim is made that the box as a whole will move
and
2. The claim is made that the magnitude of the thrust causing this movement exceeds the maximum expected thrust of the equivalent photon rocket by orders of magnitude.

Thus any successful attempt at explanation will not use conventional physics.


I fully agree with you that this is the problem: whether the EM Drive will move.  (The problem is not whether there is power flowing into the cavity, and it has a non-zero Poynting vector, or whether there are evanescent waves in the cavity, all those things are straightforward).

All these explanations posit a gradient:

Shawyer: a gradient of group velocity
Notsoureofit: a gradient of dispersion
McCulloch: a gradient of Unruh radiation wavelength
the hypothesis that the geometrical gradient produces geometrical attenuation that results in evanescent waves

and that in order to satisfy conservation of momentum, under that imposed gradient, the EM Drive has to move. 

But none of these hypothesis have been mathematically proven.  If they can be proven mathematically, and if they model the actual physical phenomena then they do not involve unconventional physics, it just an unintuitive result.

If, on the other hand, the EM Drive is an experimental artifact, or the hypotheses are not the right explanation, then they are plain wrong.  Being wrong is not unconventional physics.  Being wrong is just being wrong, conventionally speaking. ;)
« Last Edit: 06/02/2015 07:15 pm by Rodal »

Offline phaseshift

  • Full Member
  • *
  • Posts: 104
  • Seattle, WA
  • Liked: 84
  • Likes Given: 97
As many have said before the EW drive just can't work without violating CoM.  ...
Who has said that "the EM Drive drive just can't work without violating CoM"? I don't know of any author that has advocated that the EM Drive works by violating Conservation of Momentum.  Every single theory I know of, Shawyer, McCulloch, Yang, White, you name it, postulates that the EM Drive does not violate conservation of momentum (they all have different explanations as to why momentum is conserved).
I think you can find the forum is full of extremely knowledgeable people who have said as such.  Yes, the authors you listed have made claims but in each case have opened their system such that CoM appears to not be violated and that's how they make the 'claim' they are not violating basic physics.

Focusing on the question I asked might be more productive.  How did Yang open her system such that CoM does not appear to be violated?  Or do you really believe that the EM Drive, a closed cavity, is not violating CoM without a mechanism for opening it up and therefore in your opinion this line of questioning can lead no where?

Even the most disruptive force in our known universe, a black hole, it has to play by the rules and not violate CoM and CoE, drop down to plank scale you might pull something from the quantum foam, but only for the briefest amount of time. Even SpaceTime when inflation  ruled and everything accelerated away exponentially from everything else, still didn't violate CoE and CoM. To me a fundamental fact, is you can't do it. Just because it looks like the drive is a closed container it doesn't mean it really is, does it? Somehow that mix of RF bouncing around in the copper Frustum is making it to the outside world or an outside force is felt through the copper walls and the wave functions are acting on it.
Data, we need more Data, to be able to fill in the theories.

I 100% agree with you! Great way of putting it.
"It doesn't have to be a brain storm, a drizzle will often do" - phaseshift

Offline phaseshift

  • Full Member
  • *
  • Posts: 104
  • Seattle, WA
  • Liked: 84
  • Likes Given: 97
In conventional physics, it is clear that the momentum of the radiation injected into the cavity produces a back reaction upon the source of that radiation. Therefore if you put a box around [source + cavity], where the source includes a portable power supply (e.g. battery) - then the net momentum of the system is zero forever as viewed from outside the box.

If there is a radiation leak, thrust no greater than P/c ("equivalent photon rocket") may result from such a system, including thermal effects originating from the input power P.

Clearly here the claim being made is not conventional physics because:

1. The claim is made that the box as a whole will move
and
2. The claim is made that the magnitude of the thrust causing this movement exceeds the maximum expected thrust of the equivalent photon rocket by orders of magnitude.

Thus any successful attempt at explanation will not use conventional physics.


There is a corollary to all this:

If you attempt to explain this effect with conventional physics alone, you have made an error.

Thank you, thank  you, thank you.  Exactly.
"It doesn't have to be a brain storm, a drizzle will often do" - phaseshift

Offline deltaMass

  • Full Member
  • ****
  • Posts: 955
  • A Brit in California
  • Liked: 671
  • Likes Given: 275
Quote from: WarpTech
K ~ exp[2GM/r*c^2]
So K ~ 1. What use, then, is it?

Offline CW

  • Full Member
  • *
  • Posts: 188
  • Germany
  • Liked: 141
  • Likes Given: 51
As many have said before the EW drive just can't work without violating CoM.  ...
Who has said that "the EM Drive drive just can't work without violating CoM"? I don't know of any author that has advocated that the EM Drive works by violating Conservation of Momentum.  Every single theory I know of, Shawyer, McCulloch, Yang, White, you name it, postulates that the EM Drive does not violate conservation of momentum (they all have different explanations as to why momentum is conserved).

I think you can find the forum is full of extremely knowledgeable people who have said as such.  Yes, the authors you listed have made claims but in each case have opened their system such that CoM appears to not be violated and that's how they make the 'claim' they are not violating basic physics.

Focusing on the question I asked might be more productive.  How did Yang open her system such that CoM does not appear to be violated?  Or do you really believe that the EM Drive, a closed cavity, is not violating CoM without a mechanism for opening it up and therefore in your opinion this line of questioning can lead no where?

I think that the issue with closed systems is the reason that Paul March proposed a higher-dimensional space mechanism. What looks like a closed system in our 3+1 dimensional everyday reality, could easily be an open system in 5D+ spacetime. And this is why the experimentational factor is so vital. We need a demonstrator with high enough unidirectional force outout that can be reliably reproduced by any random, sufficiently technically skilled person on this planet. As string theories predict, reality is actually a lot higher-dimensional than what we can perceive and conceive with these little brains of ours.
:)
Reality is weirder than fiction

Offline phaseshift

  • Full Member
  • *
  • Posts: 104
  • Seattle, WA
  • Liked: 84
  • Likes Given: 97

I think that the issue with closed systems is the reason that Paul March proposed a higher-dimensional space mechanism. What looks like a closed system in our 3+1 dimensional everyday reality, could easily be an open system in 5D+ spacetime. And this is why the experimentational factor is so vital. We need a demonstrator with high enough unidirectional force outout that can be reliably reproduced by any random, sufficiently technically skilled person on this planet. As string theories predict, reality is actually a lot higher-dimensional than what we can perceive and conceive with these little brains of ours.
:)

There just doesn't seem to be any way around it in my mind.  Perhaps this is the first experimental evidence of a 5D spacetime (or interacting with the QV) - otherwise the thrust is an experimental artifact. I just don't see how it can be any other way. I do 'believe' there is thrust and it's not an experimental artifact - I also believe the theories proposed by Shawyer and Yang are wrong and perhaps White is on the right track - Shawyer's and Yang's drives just happen to work based on entirely different principles than they proposed.


« Last Edit: 06/02/2015 07:21 pm by phaseshift »
"It doesn't have to be a brain storm, a drizzle will often do" - phaseshift

Offline SeeShells

  • Senior Member
  • *****
  • Posts: 2442
  • Every action there's a reaction we try to grasp.
  • United States
  • Liked: 3186
  • Likes Given: 2708
I think that the issue with closed systems is the reason that Paul March proposed a higher-dimensional space mechanism. What looks like a closed system in our 3+1 dimensional everyday reality, could easily be an open system in 5D+ spacetime. And this is why the experimentational factor is so vital. We need a demonstrator with high enough unidirectional force outout that can be reliably reproduced by any random, sufficiently technically skilled person on this planet. As string theories predict, reality is actually a lot higher-dimensional than what we can perceive and conceive with these little brains of ours.
:)
Not so sure about that little brain, mine went through a period of exponential inflation, then I got onto this forum and then went through a longer period of deflation. Is there such a thing as negative mass?  :o

Offline WarpTech

  • Full Member
  • ****
  • Posts: 1407
  • Do it!
  • Statesville, NC
  • Liked: 1453
  • Likes Given: 1925
Quote from: WarpTech
K ~ exp[2GM/r*c^2]
So K ~ 1. What use, then, is it?

Uhmm.... (1/2)*Grad(K) = g/c^2  for example of using it to find the acceleration. What use is the metric tensor, g_uv?

It is an engineering tool because it is much easier working with K than it is working with g_uv and Einstein's equations, and it can be expressed in a frequency & bandwidth dependent way very easily. :)

I updated my previous post on this topic. Check it out.
http://forum.nasaspaceflight.com/index.php?topic=37642.msg1383566#msg1383566


« Last Edit: 06/02/2015 07:36 pm by WarpTech »

Offline WarpTech

  • Full Member
  • ****
  • Posts: 1407
  • Do it!
  • Statesville, NC
  • Liked: 1453
  • Likes Given: 1925

I think that the issue with closed systems is the reason that Paul March proposed a higher-dimensional space mechanism. What looks like a closed system in our 3+1 dimensional everyday reality, could easily be an open system in 5D+ spacetime. And this is why the experimentational factor is so vital. We need a demonstrator with high enough unidirectional force outout that can be reliably reproduced by any random, sufficiently technically skilled person on this planet. As string theories predict, reality is actually a lot higher-dimensional than what we can perceive and conceive with these little brains of ours.
:)

There just doesn't seem to be any way around it in my mind.  Perhaps this is the first experimental evidence of a 5D spacetime (or interacting with the QV) - otherwise the thrust is an experimental artifact. I just don't see how it can be any other way. I do 'believe' there is thrust and it's not an experimental artifact - I also believe the theories proposed by Shawyer and Yang are wrong and perhaps White is on the right track - Shawyer's and Yang's drives just happen to work based on entirely different principles than they proposed.

White is definitely NOT on the right track. Yang is closer than anyone and simply didn't disclose the details, and used a poorly constructed example to explain it, but her force equation is correct "IFF" you plug in the correct field amplitudes AND phases.

Simply put, it moves because inside there is an EM stress energy tensor that is skewed to one side by constructive and destructive interference with the source. It would have a lot more thrust if the balance were 50/50, but as it is, only a small percentage of the waves walk out of phase to cause the interference. So the standing waves dominate, and there is very little thrust. The percentage results in higher NET thrust for higher Q, but it would be a more efficient thruster if the percentage were higher and the Q were lower.

Todd

Offline sghill

  • Full Member
  • ****
  • Posts: 1682
  • United States
  • Liked: 2092
  • Likes Given: 3200
In conventional physics, it is clear that the momentum of the radiation injected into the cavity produces a back reaction upon the source of that radiation. Therefore if you put a box around [source + cavity], where the source includes a portable power supply (e.g. battery) - then the net momentum of the system is zero forever as viewed from outside the box.

If there is a radiation leak, thrust no greater than P/c ("equivalent photon rocket") may result from such a system, including thermal effects originating from the input power P.

Clearly here the claim being made is not conventional physics because:

1. The claim is made that the box as a whole will move
and
2. The claim is made that the magnitude of the thrust causing this movement exceeds the maximum expected thrust of the equivalent photon rocket by orders of magnitude.

Thus any successful attempt at explanation will not use conventional physics.


There is a corollary to all this:

If you attempt to explain this effect with conventional physics alone, you have made an error.

Thank you, thank  you, thank you.  Exactly.

But we've covered all of this literally dozens of times up thread.  Pretty please, don't rehash what we've already covered ad nauseum, and instead focus on what we haven't covered so that the thread doesn't loose meaning or focus.  Every few weeks this thread has a collective breather when someone yells "Unicorns don't exist!" then the lurkers ask a bunch of newbie questions, and then we go back to researching while the audience listens quietly. :)
« Last Edit: 06/02/2015 07:59 pm by sghill »
Bring the thunder!

Tags:
 

Advertisement NovaTech
Advertisement Northrop Grumman
Advertisement
Advertisement Margaritaville Beach Resort South Padre Island
Advertisement Brady Kenniston
Advertisement NextSpaceflight
Advertisement Nathan Barker Photography
0